Solution to Problem 692 | Beam Deflection by Method of Superposition

Problem 692
Find the value of EIδ midway between the supports for the beam shown in Fig. P-692. (Hint: Combine Case No. 11 and one half of Case No. 8.)
 

overhang beam

 

Solution 692